GMAT Exam  >  GMAT Tests  >  GMAT Focus Edition Mock test - 1 - GMAT MCQ

GMAT Focus Edition Mock test - 1 - GMAT MCQ


Test Description

30 Questions MCQ Test - GMAT Focus Edition Mock test - 1

GMAT Focus Edition Mock test - 1 for GMAT 2024 is part of GMAT preparation. The GMAT Focus Edition Mock test - 1 questions and answers have been prepared according to the GMAT exam syllabus.The GMAT Focus Edition Mock test - 1 MCQs are made for GMAT 2024 Exam. Find important definitions, questions, notes, meanings, examples, exercises, MCQs and online tests for GMAT Focus Edition Mock test - 1 below.
Solutions of GMAT Focus Edition Mock test - 1 questions in English are available as part of our course for GMAT & GMAT Focus Edition Mock test - 1 solutions in Hindi for GMAT course. Download more important topics, notes, lectures and mock test series for GMAT Exam by signing up for free. Attempt GMAT Focus Edition Mock test - 1 | 64 questions in 135 minutes | Mock test for GMAT preparation | Free important questions MCQ to study for GMAT Exam | Download free PDF with solutions
GMAT Focus Edition Mock test - 1 - Question 1

Peter invested a certain sum of money in a simple interest bond whose value grew to $300 at the end of 3 years and to $ 400 at the end of another 5 years. What was the rate of interest in which he invested his sum?

Detailed Solution for GMAT Focus Edition Mock test - 1 - Question 1

Step 1: Compute principal invested
Concept: Simple interest earned remains same year after year.

Initial amount invested = P
Value of investment (Amount) at the end of year 3 = $300
Value of investment (Amount) at the end of year 8 (another 5 years) = $400
Therefore, the interest earned for the 5 year period between the 3rd year and 8th year = $400 - $300 = $100.

So, interest earned per year = 100/5 = $20.
Therefore, interest earned for 3 years = 3 × 20 = $60.
Hence, initial amount invested P = Amount after 3 years - interest for 3 years
P = $300 − $60 = $240
Step 2: Find the rate of interest

Simple interest = $20, Principal P = $240, n = 1 year.

Rate of interest is 8.33%.

GMAT Focus Edition Mock test - 1 - Question 2

In a class 40% of the students enrolled for Math and 70% enrolled for Economics. If 15% of the students enrolled for both Math and Economics, what % of the students of the class did not enroll for either of the two subjects?

Detailed Solution for GMAT Focus Edition Mock test - 1 - Question 2

Objective: Percentage of students who enrolled for neither of the two subjects
Let A be the set of students who enrolled for Math.
Let B be the set of students who enrolled for Economics.
(A ∪ B) is the set of students who have enrolled for at least one of the two subjects.
And (A ∩ B) is the set of students who have enrolled for both Math and Economics.

n(A ∪ B) = n(A) + n(B) - n(A ∩ B)

In this question, all n(A), n(B), n(A ∪ B), and (A ∩ B) are expressed in percentage terms.

n(A ∪ B) = 40 + 70 - 15 = 95%

That is 95% of the students have enrolled for at least one of the two subjects Math or Economics.

Therefore, the balance (100 - 95)% = 5% of the students have not enrolled for either of the two subjects.

Choice A is the correct answer.

1 Crore+ students have signed up on EduRev. Have you? Download the App
GMAT Focus Edition Mock test - 1 - Question 3

If the curve described by the equation y = x2 + bx + c cuts the x-axis at -4 and y axis at 4, at which other point does it cut the x-axis?

Detailed Solution for GMAT Focus Edition Mock test - 1 - Question 3

Step 1: Understand Quadratic Equations - Parabola Theory and compute 'c'

y = x2 + bx + c is a quadratic equation and the equation represents a parabola.
The curve cuts the y-axis at 4.
The x coordinate of the point where it cuts the y-axis = 0.
Therefore, (0, 4) is a point on the curve and will satisfy the equation.
Substitute y = 4 and x = 0 in the quadratic equation: 4 = 02 + b(0) + c
Or c = 4.

Step 2: Relation between product of roots and 'c'

The product of the roots of a quadratic equation is c/a
In this question, the product of the roots = 4/1 = 4.

Step 3: What do the roots of the quadratic equation represent on the parabola?

The roots of a quadratic equation are the points where the curve (parabola) cuts the x-axis.
The question states that one of the points where the curve cuts the x-axis is -4.
So, -4 is one of roots of the quadratic equation.
Let the second root of the quadratic equation be r2.
From step 2, we know that the product of the roots of this quadratic equation is 4.
So, -4 * r2 = 4
or r2 = -1.

The second root is the second point where the curve cuts the x-axis, which is -1.

The correct choice is (A) and the correct answer is -1.

GMAT Focus Edition Mock test - 1 - Question 4

An analysis of the monthly incentives received by 5 salesmen : The mean and median of the incentives is $7000. The only mode among the observations is $12,000. Incentives paid to each salesman were in full thousands. What is the difference between the highest and the lowest incentive received by the 5 salesmen in the month?

Detailed Solution for GMAT Focus Edition Mock test - 1 - Question 4

Hints to solve this question:

  1. Mean and Median = $7000. So, find the third highest incentive.
  2. Only one mode; mode = $12,000.
  3. Use hint 1 and hint 2 to find how many executives have got $12,000.
  4. Now compute the sum of incentives got by those who got neither $7000 nor $12000.

Step 1: Understanding the given data

  1. The arithmetic mean of the incentives is $7000.
  2. The median of the incentives is also $7000.
  3. There is only one mode and the mode is $12,000.

Step 2: Decoding Mean and Median

Let their incentives be a, b, c, d, and e such that a ≤ b ≤ c ≤ d ≤ e

Therefore, the median of these values is 'c'.
The median incentive is $7000. So, c = $7000.

Essentially, the incentives are __ __ 7000 __ __

The arithmetic mean of the incentives is $7000.
So, the sum of their incentives a + b + c + d + e = 5 * 7000 = $35,000

Step 3: Decoding Mode

There is only one mode amongst these 5 observations.
The mode is that value that appears with the maximum frequency.
Hence, $12,000 is the incentive received by the most number of salesmen.

So, the incentives are __ __ 7000, 12000, 12000

Step 4: Putting it all together

The incentive that c has got is $7000
The incentive received by d and e are 12,000 each

Therefore, c + d + e = 7000 + 12,000 + 12,000 = $31,000
Hence, a + b = 35,000 - 31,000 = $4000

As there is only one mode, the incentives received by a and b have to be different.
So, a received $1000 and b received $3000.

Maximum incentive: $12,000
Minimum incentive: $1000
Difference between maximum and minimum incentive: $11,000

Choice E is the correct answer.

GMAT Focus Edition Mock test - 1 - Question 5

What is the remainder when 1044 × 1047 × 1050 × 1053 is divided by 33?

Detailed Solution for GMAT Focus Edition Mock test - 1 - Question 5

Useful result pertaining to remainders

You can solve this problem if you know this rule about remainders.
Let a number x divide the product of A and B.
The remainder will be the product of the remainders when x divides A and when x divides B.

Using this rule,
The remainder when 33 divides 1044 is 21.
The remainder when 33 divides 1047 is 24.
The remainder when 33 divides 1050 is 27.
The remainder when 33 divides 1053 is 30.

∴ the remainder when 33 divides 1044 × 1047 × 1050 × 1053 is 21 × 24 × 27 × 30.

Note: The remainder when a number is divided by a divisor 'd' will take values from 0 to (d - 1). It will not be equal to or more than 'd'

The value of 21 × 24 × 27 × 30 is more than 33.
When the value of the remainder is more than the divisor, the final remainder will be the remainder of dividing the product by the divisor.
i.e., the final remainder is the remainder when 33 divides 21 × 24 × 27 × 30.
When 33 divides 21 × 24 × 27 × 30, the remainder is 30.

Choice C is the correct answer.

GMAT Focus Edition Mock test - 1 - Question 6

Set A contains all the even numbers between 2 and 50 inclusive. Set B contains all the even numbers between 102 and 150 inclusive. What is the difference between the sum of elements of set B and that of set A?

Detailed Solution for GMAT Focus Edition Mock test - 1 - Question 6

SET A: {2, 4, 6, 8,...., 50}. Set of first 25 consecutive positive even numbers.

SET B: {102, 104, 106,....., 150}. Another set of 25 consecutive even numbers starting from 102.

Difference between 1st term of set A and that of set B is 100. Difference between 2nd term of set A and that of set B is 100.
Each term in set B is 100 more than the corresponding term in set A.

So sum of the differences of all the terms is (100 + 100 + 100 + ....) = 25 * 100 = 2500.

Choice A is the correct answer.

GMAT Focus Edition Mock test - 1 - Question 7

Ram, who is half as efficient as Krish, will take 24 days to complete a task if he worked alone. If Ram and Krish worked together, how long will they take to complete the task?

Detailed Solution for GMAT Focus Edition Mock test - 1 - Question 7

Ram takes 24 days to complete the task, if he works alone.
Krish is twice as efficient as Ram is. So, working alone, Krish will take half the time to complete the task.i.e., 12 days.

Ram will complete 1/24th of the task in a day.
Krish will complete 1/12th of the task in a day.
When they work together, they will complete  of the task in a day.

Therefore, when they work together they will complete the task in 8 days.

Choice C is the correct answer.

GMAT Focus Edition Mock test - 1 - Question 8

What is the smallest integer that satisfies the inequality > 0?

Detailed Solution for GMAT Focus Edition Mock test - 1 - Question 8

Let us factorize the denominator and rewrite the expression as 
Approach: Equate each of the terms of the expression to zero to identify the values of x in which the inequality holds good.
The values that are relevant to us are x = 3, x = 10 and x = -2.

Let us arrange these values in ascending order: -2, 3 and 10.
The quickest way to solve inequalities questions after arriving at these values is verifying whether the inequality holds good at the following intervals.

Interval 1: x < -2.
Pick a value in that range and check whether the inequality holds good.
Let us take x = -10. When x = -10, the value of 
The value of the expression in this interval is negative; the inequality DOES NOT hold good in this interval.

Interval 2: -2 < x < 3.
Let us take x = 0. When x = 0,  the inequality holds good in this interval.
We found that the inequality holds good in the interval -2 < x < 3
The least integer value that x can take in the interval -2 < x < 3 is x = -1.

So, the correct answer is -1.

Remember: We have to find out the least integer value. And we have arrived at -1.
Do not waste time computing the entire range of values of x that satisfy the inequality.
Note: In any inequality question, when the question asks us to determine the intervals in which the inequality holds good, we have to eliminate values of x that will result in the denominator becoming zero.

Choice D is the correct answer.

GMAT Focus Edition Mock test - 1 - Question 9

Ann invested a certain sum of money in a bank that paid simple interest. The amount grew to $240 at the end of 2 years. She waited for another 3 years and got a final amount of $300. What was the principal amount that she invested at the beginning?

Detailed Solution for GMAT Focus Edition Mock test - 1 - Question 9

Concept: Simple interest earned remains same year after year.

At the end of 2 years: Value of investment (Amount) = $240
At the end of another 3 years: Value of investment (Amount) = $300
In 3 years, the sum grew by $60.
So interest for 3 years = $60.
Because Ann invested in simple interest, interest earned each year = $20.

Interest earned in the first 2 years = 2 × 20 = $40.
Therefore, principal = Sum at the end of 2 years - $40 = $240 - $40 = $200.
Principal invested $200

Choice A is the correct answer.

GMAT Focus Edition Mock test - 1 - Question 10

How many real solutions exist for the equation x2 – 11|x| - 60 = 0?

Detailed Solution for GMAT Focus Edition Mock test - 1 - Question 10

Step 1: Assign y = |x| and solve for y

Let |x| = y.
We can rewrite the equation x2 - 11|x| - 60 = 0 as y2 - 11y - 60 = 0
The equation can be factorized as y2 - 15y + 4y - 60 = 0
(y - 15) (y + 4) = 0
The values of y that satisfy the equation are y = 15 or y = -4.

Step 2: Compute number of real values of x

We have assigned y = |x|
|x| is always a non-negative number.
So, |x| cannot be -4.
|x| can take only one value = 15.
If |x| = 15, x = 15 or -15.

The number of real solutions that exist for x2 – 11|x| - 60 = 0 is 2.

The correct choice is (B) and the correct answer is 2.

GMAT Focus Edition Mock test - 1 - Question 11

How many trailing zeros will be there after the rightmost non-zero digit in the value of 25!?

Detailed Solution for GMAT Focus Edition Mock test - 1 - Question 11

25! means factorial 25 whose value = 25 × 24 × 23 × 22 × .... × 1

When a number that is a multiple of 5 is multiplied with an even number, it results in a trailing zero.
(Product of 5 and 2 is 10 and any number when multiplied with 10 or a power of 10 will have one or as many zeroes as the power of 10 with which it has been multiplied)

In 25!, the following numbers have 5 as their factor: 5, 10, 15, 20, and 25.
25 is the square of 5 and hence it has two 5s in it.
In toto, it is equivalent of having six 5s.

There are at least 6 even numbers in 25!
Hence, the number 25! will have 6 trailing zeroes in it.

Choice C is the correct answer.

GMAT Focus Edition Mock test - 1 - Question 12

A can complete a project in 20 days and B can complete the same project in 30 days. If A and B start working on the project together and A quits 10 days before the project is completed, in how many days will the project be completed?

Detailed Solution for GMAT Focus Edition Mock test - 1 - Question 12

A can complete a project in 20 days. So, A will complete 1/20th of the project in a day.
B can complete a project in 30 days. So, B will complete 1/30th of the project in a day.

Let the total number of days taken to complete the project be 'x' days.
The value of x is the answer to the question.

B worked all x days. However, A worked for (x - 10) days because A quits 10 days before the project is completed.

In a day, A completes 1/20th of the project.
Therefore, A would have completed  of the project in (x - 10) days.

In a day, B completes 1/30th of the project.
Therefore, B would have completed x/30th of the project in x days.

3x - 30 + 2x = 60
5x = 90 or x = 18

Choice A is the correct answer.

GMAT Focus Edition Mock test - 1 - Question 13

What range of values of 'x' will satisfy the inequality 

Detailed Solution for GMAT Focus Edition Mock test - 1 - Question 13

We can rewrite the above inequality as

Split the middle term of the numerator as a precursor to factorizing it: 

Factorize the quadratic expression: 
The above inequality will hold good if the numerator and denominator are both positive or are both negative.

Possibility 1: When (5x - 2)(3x + 1) > 0 and x > 0
Rule: (x - a)(x - b) > 0 when x does not lie between "a" and "b".
Applying the rule, the values of 'x' that will satisfy (5x - 2)(3x + 1) > 0 will not lie between
Combining the above result with the second condition that x > 0, we get x > 2/5

Possibility 2: When (5x - 2)(3x + 1) < 0 and x < 0.
Rule: (x - a)(x - b) < 0 when x lies between "a" and "b".
Applying the rule, the following values of 'x' will satisfy (5x - 2)(3x + 1) < 0:
Combining the above range of values with the second condition that x < 0, we get

Therefore, the range of values of x that will satisfy the inequality is:

Choice D is the correct answer.

GMAT Focus Edition Mock test - 1 - Question 14

Shawn invested one half of his savings in a bond that paid simple interest for 2 years and received $ 550 as interest. He invested the remaining in a bond that paid compound interest, interest being compounded annually, for the same 2 years at the same rate of interest and received $605 as interest. What was the value of his total savings before investing in these two bonds?

Detailed Solution for GMAT Focus Edition Mock test - 1 - Question 14

Step 1: Compute interest for each year for both simple and compound interest

Simple Interest:
Concept: Simple interest earned is same value year on year.
Shawn received $550 as interest for 2 years.
Simple interest earned for first year = 550/2 = $275
The simple interest for second year is also $275.

In Compound interest: Shawn received $605 as interest for 2 years.
Concept: Interest earned is same for both simple and compound interest in the first year.
Compund interest earned for first year = $275.
Compund interest earned for second year = 605 − 275 = $330
Extra interest received from compound interest = $55.
Interest for each year

Step 2: Find the rate of interest

Concept: In Compound interest, interest earned on first year's interest will get added in second year and contributes to the additional interest when invested in compound interest.

Compound interest for first year = $275.
Therefore, $55 is the interest earned during the second year on $275.
Therefore, the rate of interest = 

Step 3: Compute the principal invested

At 20% rate of interest, the simple interest earned for 1 year = $275


or P = $1375

Shawn had invested equal sums in both the bonds.
His total savings before investing = 2 × 1375 = $2750
Total savings = $2750

Choice D is the correct answer.

GMAT Focus Edition Mock test - 1 - Question 15

How many keystrokes are needed to type numbers from 1 to 1000?

Detailed Solution for GMAT Focus Edition Mock test - 1 - Question 15

While typing numbers from 1 to 1000, there are 9 single digit numbers: from 1 to 9.
Each of these numbers requires one keystroke.
That is 9 key strokes.

There are 90 two-digit numbers: from 10 to 99.
Each of these numbers requires 2 keystrokes.
Therefore, 180 keystrokes to type the 2-digit numbers.

There are 900 three-digit numbers: from 100 to 999.
Each of these numbers requires 3 keystrokes.
Therefore, 2700 keystrokes to type the 3-digit numbers.

1000 is a four-digit number which requires 4 keystrokes.

Totally, therefore, one requires 9 + 180 + 2700 + 4 = 2893 keystrokes.
Choice B is the correct answer.

Watch out for the common mistake that many of us make of counting only 89 2-digit numbers and 899 3-digit numbers. The temptation is to say, 99 - 10 = 89. So, 89 2-digit numbers exist. 99 - 10 means that we are not counting 10 as a 2-digit number. The correct approach is: of the 99 numbers from 1 to 99, we are not counting the first 9 single digit numbers. So, we have 99 - 9 = 90 2-digit numbers. The same logic applies when we count 3-digit numbers.

GMAT Focus Edition Mock test - 1 - Question 16

If "x" is an integer, which of the following inequalities has (have) a finite range of values of "x" satisfying it (them)?

Detailed Solution for GMAT Focus Edition Mock test - 1 - Question 16

Step 1: Find the values of "x" that will satisfy the four inequalities

Choice A: x2 + 5x + 6 > 0
Factorize the given expression: x2 + 5x + 6 > 0 = (x + 2)(x + 3) > 0.
This inequality will hold good when both (x + 2) and (x + 3) are simultaneously positive OR are simultaneously negative.

Possibility 1: Both (x + 2) and (x + 3) are positive.
i.e., x + 2 > 0 AND x + 3 > 0
i.e., x > -2 AND x > -3
Essentially translates to x > -2

Possibility 2: Both (x + 2) and (x + 3) are negative.
i.e., x + 2 < 0 AND x + 3 < 0
i.e., x < -2 AND x < -3
Essentially translates to x < -3

Evaluating both the possibilities, we get the range of values of "x" that satisfy this inequality to be x > -2 or x < -3. i.e., "x" does not lie between -3 and -2.
i.e., x takes values lesser than -3 or greater than -2.
The range of values that x takes is infinite.

Choice B: |x + 2| > 4
|x + 2| > 4 is a modulus function and therefore, has two possibilities

Possiblity 1: x + 2 > 4
i.e., x > 2

Possiblity 2: (x + 2) < -4.
i.e., x < -6
Evaluating the two options together, we get the values of "x" that satisfy the inequality as x > 2 OR x < -6.
i.e., "x" does not lie between -6 and 2.
An infinite range of values.

Choice C: 9x - 7 < 3x + 14
Simplifying, we get 6x < 21 or x < 3.5.
An infinite range of values.

Choice D: x2 - 4x + 3 < 0
Factorizing x2 - 4x + 3 < 0 we get, (x - 3)(x - 1) < 0.
This inequality will hold good when one of the terms (x - 3) or (x - 1) is positive and the other is negative.

Possibility 1: (x -3) is positive and (x - 1) is negative.
i.e., x - 3 > 0 AND x -1 < 0
i.e., x > 3 AND x < 1
Such a number DOES NOT exist. It is an infeasible solution.

Possibility 2: (x - 3) is negative and (x - 1) is positive.
i.e., x - 3 < 0 AND x - 1 > 0
i.e., x < 3 AND x > 1
Essentially translates to 1 < x < 3 Finite range of values for "x".

Choice D is the correct answer.

GMAT Focus Edition Mock test - 1 - Question 17

Braun invested a certain sum of money at 8% p.a. simple interest for 'n' years. At the end of 'n' years, Braun got back 4 times his original investment. What is the value of n?

Detailed Solution for GMAT Focus Edition Mock test - 1 - Question 17

Step 1: Assume initial investment (Principal) and compute final value (amount) after n years


For any assumed principal, the number of years is going to remain the same because the amount is expressed as ‘x’ times the principal i.e., 4 times in this case.
Let us assume the initial investment (principal) by Braun to be $100.
Amount = 4 × Principal = $400
Amount = Principal + Simple Interest
Therefore, the Simple Interest earned = 400 - 100 = $300.

Step 2: Find the number of years n

Substitute assumed value of principal and the corresponding interest earned, and rate of interest in equation 1.

Or 8n = 300
Or n = 37.5 years
Any amount, when invested for 37.5 years at 8% per annum simple interest would become 4 times the initial value.

Alternative Method:

Find the number of years required to double the initial amount (principal).
When the initial investment doubles, the interest earned is the same as the initial investment (principal).
So, if principal = 100, interest earned = 100 and r = 8%.

So, n = 100/8 = 12.5 years.

Initial investment of $100 becomes $400 after earning an interest of $300.
To earn $100 interest it took 12.5 years. Hence, it will take 3 × 12.5 = 37.5 years to earn $300 interest.

Choice D is the correct answer.

GMAT Focus Edition Mock test - 1 - Question 18

How many integral divisors does the number 120 have?

Detailed Solution for GMAT Focus Edition Mock test - 1 - Question 18

Step 1: Express the number in terms of its prime factors

120 = 23 * 3 * 5.
The three prime factors are 2, 3 and 5.
The powers of these prime factors are 3, 1 and 1 respectively.

Step 2: Find the number of factors as follows

To find the number of factors / integral divisors that 120 has, increment the powers of each of the prime factors by 1 and then multiply them.

Number of factors = (3 + 1) * (1 + 1) * (1 + 1) = 4 * 2 * 2 =16

Choice B is the correct answer.

Key Takeaway
How to find the number of factors of a number? Method: Prime Factorization
Let the number be 'n'.
Step 1: Prime factorize 'n'. Let n = ap * bq, where 'a' and 'b' are the only prime factors of 'n'.
Step 2: Number of factors equals product of powers of primes incremented by 1.
i.e., number of factors = (p + 1)(q + 1)

GMAT Focus Edition Mock test - 1 - Question 19

When 242 is divided by a certain divisor the remainder obtained is 8. When 698 is divided by the same divisor the remainder obtained is 9. However, when the sum of the two numbers 242 and 698 is divided by the divisor, the remainder obtained is 4. What is the value of the divisor?

Detailed Solution for GMAT Focus Edition Mock test - 1 - Question 19

When 242 is divided by a certain divisor the remainder obtained is 8.

Let the divisor be d.
When 242 is divided by d, let the quotient be 'x'. The remainder is 8.
Therefore, 242 = xd + 8

When 698 is divided by the same divisor the remainder obtained is 9.

Let y be the quotient when 698 is divided by d.
Then, 698 = yd + 9.

When the sum of the two numbers, 242 and 698, is divided by the divisor, the remainder obtained is 4.

242 + 698 = 940 = xd + yd + 8 + 9
940 = xd + yd + 17

Because xd and yd are divisible by d, the remainder when 940 is divided by d should have been 17.

However, because we know that the remainder is 4, it would be possible only when 17/d leaves a remainder of 4.

If the remainder obtained is 4 when 17 is divided by 'd', then 'd' has to be 13.

Choice C is the correct answer.

GMAT Focus Edition Mock test - 1 - Question 20

A merchant who marked his goods up by 50% subsequently offered a discount of 20% on the marked price. What is the percentage profit that the merchant make after offering the discount?

Detailed Solution for GMAT Focus Edition Mock test - 1 - Question 20

Step 1: Assume cost price and find marked price

Approach: Assume cost price as $100.

Let the cost price of the goods to be $ 100.
The merchant marks his goods up by 50%.
Therefore, his marked price = cost price + mark up.
Marked price = $100 + 50% of $100 = 100 + 50 = $150.

Step 2: Find the discount offered and Selling price

The merchant offers a discount of 20% on the marked price.
Discount offered = 20% of 150 = $30.
Therefore, the goods are finally sold at $150 − $30 = $ 120.

Cost price is $100 and final selling price is $120.
Therefore, profit made = $20 on the cost price of $ 100.

Merchant made a profit of 20%

Choice D is the correct answer.

GMAT Focus Edition Mock test - 1 - Question 21

A number when divided by a divisor leaves a remainder of 24. When twice the original number is divided by the same divisor, the remainder is 11. What is the value of the divisor?

Detailed Solution for GMAT Focus Edition Mock test - 1 - Question 21

Step 1: Decode "A number when divided by a divisor leaves a remainder of 24"

Let the original number be 'a'.
Let the divisor be 'd'.
Let the quotient of dividing 'a' by 'd' be 'x'.
Therefore, we can write the division as a/d = x and the remainder is 24.
i.e., a = dx + 24

Step 2: Decode "When twice the original number is divided by the same divisor, the remainder is 11"

Twice the original number is divided by 'd' means 2a is divided by d.
We know from Step 1 that a = dx + 24.
Therefore, 2a = 2(dx + 48) or 2a = 2dx + 48

When (2dx + 48) is divided by 'd' the remainder is 11.
2dx is divisible by 'd' and will therefore, not leave a remainder.
The remainder of 11 would be the remainder of dividing 48 by d.

The question essentially becomes "What number will leave a remainder of 11 when it divides 48?"
When 37 divides 48, the remainder is 11.

Hence, the divisor is 37.

Choice D is the correct answer.

GMAT Focus Edition Mock test - 1 - Question 22

In the twentieth century, the visual arts have embarked on major experimentation, from cubism to expressionism.  While tastes always vary, there are certainly some people who find beautiful objects of each of the art movements of the first half of the twentieth century.  In the latter half of the twentieth century, though, most works are so abstract or shocking that neither the critic nor the general public uses the word "beautiful" to describe them: indeed, sometimes late twentieth-century artists have, as one of their expressed goals, the creation of a work that no one could find beautiful.  Whatever these artists are creating may be intellectually engaging at some level, but it is no longer art.

Q. Which of the following is an assumption that supports drawing the conclusion above from the reasons given for that conclusion?

Detailed Solution for GMAT Focus Edition Mock test - 1 - Question 22

The argument makes a number of factual statements.  Art in the first half of the 20th century are, or could be considered, beautiful.  Works by artists in the latter half of the 20th century are not supposed to be beautiful, and even, are supposed to be devoid of beauty. Then the argument draws a bold powerful conclusion: therefore, they are not art!  The assumption seems to be something that links beauty to whether something qualifies as art.  We definitely need an answer to speak to the question: what does, or doesn't, qualify as art? 

(C) is credited answer.  If something needs to be beautiful, or potentially beautiful, to qualify as art, then this would explain that works that "no one could find beautiful" would fall outside the author's definition of art. 

The other answers are all quite tempting, because we could imagine an art professor or someone in an art class arguing for any one of them. 

(A) is irrelevant. Critics & the general public might have different appraisals, but what one or the other thinks does not, in and of itself, seem to determine whether something is art. 

(B) is also irrelevant: who determines the meaning is a separate question from whether the work qualifies at art in the first place.  (BTW, exceedingly few modern critics would accept the interpretive idea contained in choice .) 

(D) is undeniably true, but not relevant: again: it provides no standard by which we could say the former objects are art and the latter objects aren’t. 

(E) is a far-flung idea, unrelated to the discussion. The passage doesn't address the issue of whether any works of art are intellectually engaging.

GMAT Focus Edition Mock test - 1 - Question 23

In a few recent cases, some teenagers with advanced programming abilities used a new programming language, FANTOD, to hack into ETS and change their own SAT scores. All of the teenagers convicted of this crime were highly skilled in programming FANTOD. In light of these cases, some colleges have discounted the official SAT scores of applicants with a knowledge of FANTOD, and have required them to take special admission tests in supervised conditions on their own campuses.

Q. Which of following conclusions can most properly be drawn from the information above?

Detailed Solution for GMAT Focus Edition Mock test - 1 - Question 23

The evidence says: all the ETS hackers were FANTOD programmers. What the colleges seem to be assuming is the converse: all FANTOD programmers are hackers.  Of course, there is no direct evidence for this converse. Presumably there are some students who learn FANTOD in good faith and who are not hackers, but because of the assumption the colleges are making, these students are faced with extra challenges, such as having their justly achieved SAT scores disregarded and being forced to take additional admission tests. 

(C) is the credited answer. Since there is no evidence for the converse statement, we have reason to believe there are FANTOD programmers who are entirely innocent of any hacking, yet those very students will have their perfectly valid SAT scores dismissed and will have to take a new test to achieve admission: this certainly would not be fun, would not be fair, and could place them at a disadvantage with respect to all the non-programming students who could just take the ordinary SATs and be done with all testing. 

(A) assumes too much based on the information provided in the prompt. Specifically, we only know about a specific group of those with FANTOD knowledge: those who used it to hack into ETS. Therefore, we cannot make any airtight conclusions about "most people". It is very possible that most people who know FANTOD use it for purposes other than hacking.  

(B) might be true, but it's much too broad. This is about the much larger issue of what is the best way for colleges to determine who should be admitted.  This entire argument is focused quite specifically on the FANTOD programmers and the issues associated with them. 

We have absolutely no evidence for (D). All we know is that, whatever scores those hackers achieved on the real SAT, they falsified the records to make them higher. We don't know if those scores were already high, and we certainly can draw no conclusion about all the students who know how to program in FANTOD who are not hackers. In fact, one might suspect the opposite, that folks bright enough to figure out this sophisticated programming language might be more intelligent and more successful on average, but even that we strictly can't assume. Therefore, we can't draw a clear conclusion about this. 

(E) is a tricky one. We are told that some colleges took a certain set of special measures. We are given no information on what the other colleges did. Did they take another set of special measures? Did they not address the issue at all?  We don't know. Therefore, we can't draw a clear conclusion along these lines.

GMAT Focus Edition Mock test - 1 - Question 24

A minor league baseball franchise experienced a drop in attendance this week after they suffered three losses by margins of ten runs or more last week.  Many spectators of those games wrote letters to the editors of the local sporting news, complaining of the poor play of the team in those three losses. Nevertheless, the front office of this baseball franchise maintains that the team's poor play in those three losses has nothing to do with this week's decline in attendance.

Q. Which of the following, if true, most strongly supports the position held by the front office of the baseball franchise?

Detailed Solution for GMAT Focus Edition Mock test - 1 - Question 24

The statement by the front office of the baseball franchise seems, on the surface, not to take the basic facts into account.  If we want to strengthen this position, there must be some alternate explanation for the drop in attendance. 

Part of strengthening the franchise's position would be weakening the original position: namely, that the team's poor play explains the drop in attendance. 

(C) is the credited answer. If other minor league teams also experience a drop this week, there must be something global in this market affecting all teams. We don't know what this factor is, but it's something that touches all teams, not just those that played badly last week. This provides a cogent alternative explanation, even though we don't know the specific nature of the factor causing the drop in attendance. 

Both (A) & (D) strengthen the original position, namely, that the team's poor play explains the drop in attendance. In order to strengthen the baseball franchise's position, we have to weaken this original position. 

Choice (B) essentially accuses the baseball franchise of lying, or at least bluffing, which hardly strengthens their position. 

Choice (E) simply adds to the paradox: if the closest MLB team is far away and folks typically don't go there, then there would be more demand for the local minor league baseball. Given that demand, a drop in attendance doesn't make as much sense.  This choice adds to the confusion without explaining anything.

GMAT Focus Edition Mock test - 1 - Question 25

Megalimpet is a nationwide owner of office space. They have major office buildings in the downtowns of several cities in the 48 lower states, and rent this space to individual companies. Megalimpet office spaces vary from small office to large suites, and every space has custom-designed wall-to-wall carpeting. The carpet in several Megalimpet facilities needed replacing. The winning bid for the nationwide carpet replacement was submitted by Bathyderm Carpet Company (BCC). The bid contract involves all delivery costs, all installation, and any ongoing maintenance and upkeep while the carpet is under the three-year warranty. Both BCC executives and independent consultants they hired felt BCC would be able to perform all these services for far less than their bid price; these circumstances would allow BCC to reap a considerable profit.

Q. Which of the following, if true, most calls in question the argument that BCC will make a large profit from this contract with Megalimpet?

Detailed Solution for GMAT Focus Edition Mock test - 1 - Question 25

From all the evidence given, it seems that BCC (and the independent consultants) have taken all costs into account, and the analysis reveals that they will reap a considerable profit. In order to call this into question, we have to come up with some major unanticipated cost that would not be something already considered in this analysis.

(D) is the credited answer. First of all, laser printers and photocopiers are very common devices in office spaces, so we good reason to think that many of Megalimpet's tenants will use these. If the toner degrades the carpet, that's a huge additional expense for BCC, because their contract includes "ongoing maintenance" --- i.e. replacing any carpet that needs replacing. Finally, nothing in the argument stem gives us any indication that this problem was on anybody's radar, so this well could be an unexpected or unanticipated expense for BCC. Therefore, it most calls into question the idea that BCC will make a huge profit.

(A) & (C) are all expenses that would have been very clear to BCC and to its independent consultants, and therefore all of these would have had to have been taken into account when the financial analysis of the bid was made. There is no reason any of these expenses would be unanticipated.

(B) speaks to BCC previous experience, which, if anything, would tend to suggest they know what they are talking about. If anything, this would tend to strengthen the argument, not weaken it.

(E) only compares BCC to the second lowest bid, but we have no idea about that company, what it did or did not take into account in their bid, and what their overall costs might be. There are too many unknowns for this piece of information, by itself, to have any substantial impact on the argument.

GMAT Focus Edition Mock test - 1 - Question 26

Direction: Read the following Passage and Answer the following Question.

“Strange Bedfellows!” lamented the title of a recent letter to Museum News, in which a certain Harriet Sherman excoriated the National Gallery of Art in Washington for its handling of tickets to the much-ballyhooed “Van Gogh’s van Goghs” exhibit. A huge proportion of the 200,000 free tickets were snatched up by the opportunists in the dead of winter, who then scalped those tickets at $85 apiece to less hardy connoiseurs.

Yet, Sherman’s bedfellows are far from strange. Art, despite its religious and magical origins, very soon became a commercial venture. From bourgeois patrons funding art they barely understood in order to share their protegee’s prestige, to museum curators stage-managing the cult of artists in order to enhance the market value of museum holdings, entrepreneurs have found validation and profit in big-name art. Speculators, thieves, and promoters long ago created and fed a market where cultural icons could be traded like commodities.

This trend toward commodification of high-brow art took an ominous, if predictable, turn in the 1980s during the Japanese “bubble economy.” At a time when Japanese share prices more than doubled, individual tycoons and industrial giants alike invested record amounts in some of the West’s greatest masterpieces. Ryoei Saito, for example, purchased van Gogh’s Portrait of Dr. Gachet for a record-breaking $82.5 million. The work, then on loan to the Metropolitan Museum of Modern Art, suddenly vanished from the public domain. Later learning that he owed the Japanese government $24 million in taxes, Saito remarked that he would have the paining cremated with him to spare his heirs the inheritance tax. This statement, which he later dismissed as a joke, alarmed and enraged many. A representative of the Van Gogh museum, conceding that he had no legal redress, made an ethical appeal to Mr. Saito, asserting, “a work of art remains the possession of the world at large.”

Ethical appeals notwithstanding, great art will increasingly devolve into big business. Firstly, great art can only be certified by its market value. Moreover, the “world at large” hasn’t the means of acquisition. Only one museum currently has the funding to contend for the best pieces–the J. Paul Getty Museum, founded by the billionaire oilman. The art may disappear into private hands, but its transfer will disseminate once static fortunes into the hands of various investors, collectors, and occasionally the artist.

Q. Which of the following would be the most appropriate title for the passage?

Detailed Solution for GMAT Focus Edition Mock test - 1 - Question 26

This is a GLOBAL question. A good title should sum up the theme and content of the passage as a whole. If you see a question asking you to choose a title for a passage, you have encountered a Global question and should look at the passage as a whole, using the Topic, Scope, and Purpose that you noted to help find your answer. You are looking for a choice that represents the author’s view that art and business are closely connected.

Choice (A) is a distortion of the topic. The issue of “art of art’s sake” does underlie the passage, and there is some attention to an incident involving a Japanese businessperson, but there is no suggestion that the ideal is particularly Japanese.

Choice (B) focuses on the artist van Gogh, who is mentioned in two paragraphs; however, van Gogh is not the topic of the passage, and there is no discussion of his innovations.

Choice (C) distorts the topic of the first paragraph. In fact, that paragraph discussed a letter published in a magazine, but it did not discuss the press per se. The passage does not actually state who the “strange bedfellows” were, but the implication is that Sherman was referring to either the scalpers and the art aficionados who were vying for tickets, or to art and (illegal) business.

Choice (D) is actually a reversal of the author’s theme, which is that money and are art quite often intimately linked; the first sentence of paragraph 4 dismisses the ethical concerns.

The correct answer is choice (E), which states that art is business.

GMAT Focus Edition Mock test - 1 - Question 27

Direction: Read the following Passage and Answer the following Question.

“Strange Bedfellows!” lamented the title of a recent letter to Museum News, in which a certain Harriet Sherman excoriated the National Gallery of Art in Washington for its handling of tickets to the much-ballyhooed “Van Gogh’s van Goghs” exhibit. A huge proportion of the 200,000 free tickets were snatched up by the opportunists in the dead of winter, who then scalped those tickets at $85 apiece to less hardy connoiseurs.

Yet, Sherman’s bedfellows are far from strange. Art, despite its religious and magical origins, very soon became a commercial venture. From bourgeois patrons funding art they barely understood in order to share their protegee’s prestige, to museum curators stage-managing the cult of artists in order to enhance the market value of museum holdings, entrepreneurs have found validation and profit in big-name art. Speculators, thieves, and promoters long ago created and fed a market where cultural icons could be traded like commodities.

This trend toward commodification of high-brow art took an ominous, if predictable, turn in the 1980s during the Japanese “bubble economy.” At a time when Japanese share prices more than doubled, individual tycoons and industrial giants alike invested record amounts in some of the West’s greatest masterpieces. Ryoei Saito, for example, purchased van Gogh’s Portrait of Dr. Gachet for a record-breaking $82.5 million. The work, then on loan to the Metropolitan Museum of Modern Art, suddenly vanished from the public domain. Later learning that he owed the Japanese government $24 million in taxes, Saito remarked that he would have the paining cremated with him to spare his heirs the inheritance tax. This statement, which he later dismissed as a joke, alarmed and enraged many. A representative of the Van Gogh museum, conceding that he had no legal redress, made an ethical appeal to Mr. Saito, asserting, “a work of art remains the possession of the world at large.”

Ethical appeals notwithstanding, great art will increasingly devolve into big business. Firstly, great art can only be certified by its market value. Moreover, the “world at large” hasn’t the means of acquisition. Only one museum currently has the funding to contend for the best pieces–the J. Paul Getty Museum, founded by the billionaire oilman. The art may disappear into private hands, but its transfer will disseminate once static fortunes into the hands of various investors, collectors, and occasionally the artist.

Q. It can be inferred from the passage that Harriet Sherman would be most likely to agree with which of the following statements regarding admission to museum exhibits?

Detailed Solution for GMAT Focus Edition Mock test - 1 - Question 27

This is an INFERENCE question. It’s clear from the word ‘inferred’, of course, but the phrase ‘most likely to agree with’ is also a powerful indicator that you have encountered an Inference question on the GMAT. Use the notes that you have made for Topic, Scope, and Purpose, and look for an answer choice that is directly supported by the passage.

In order to answer this Inference question, use your passage map to locate where Sherman’s argument was presented – in the first paragraph. Sherman was angry because people with a genuine interest in art were forced to pay very high prices for tickets that were supposed to be free.

Choice (A) is a 180-degree reversal of her point: it was the first-come-first-served rule that allowed opportunists to get so many tickets.

Choice (B) is a strong choice, and is supported by the fact that Sherman was angry that those with a genuine interest in art had to pay high ticket prices.

Choice (C) may be true, but it is beyond the scope of this passage.

Choices (D) and (E) represent possible solutions to the problem raised by Sherman, but there is no support in the passage that either Sherman of the author would find them satisfactory.

Choice (B) is the correct answer.

GMAT Focus Edition Mock test - 1 - Question 28

Direction: Read the following Passage and Answer the following Question.

“Strange Bedfellows!” lamented the title of a recent letter to Museum News, in which a certain Harriet Sherman excoriated the National Gallery of Art in Washington for its handling of tickets to the much-ballyhooed “Van Gogh’s van Goghs” exhibit. A huge proportion of the 200,000 free tickets were snatched up by the opportunists in the dead of winter, who then scalped those tickets at $85 apiece to less hardy connoiseurs.

Yet, Sherman’s bedfellows are far from strange. Art, despite its religious and magical origins, very soon became a commercial venture. From bourgeois patrons funding art they barely understood in order to share their protegee’s prestige, to museum curators stage-managing the cult of artists in order to enhance the market value of museum holdings, entrepreneurs have found validation and profit in big-name art. Speculators, thieves, and promoters long ago created and fed a market where cultural icons could be traded like commodities.

This trend toward commodification of high-brow art took an ominous, if predictable, turn in the 1980s during the Japanese “bubble economy.” At a time when Japanese share prices more than doubled, individual tycoons and industrial giants alike invested record amounts in some of the West’s greatest masterpieces. Ryoei Saito, for example, purchased van Gogh’s Portrait of Dr. Gachet for a record-breaking $82.5 million. The work, then on loan to the Metropolitan Museum of Modern Art, suddenly vanished from the public domain. Later learning that he owed the Japanese government $24 million in taxes, Saito remarked that he would have the paining cremated with him to spare his heirs the inheritance tax. This statement, which he later dismissed as a joke, alarmed and enraged many. A representative of the Van Gogh museum, conceding that he had no legal redress, made an ethical appeal to Mr. Saito, asserting, “a work of art remains the possession of the world at large.”

Ethical appeals notwithstanding, great art will increasingly devolve into big business. Firstly, great art can only be certified by its market value. Moreover, the “world at large” hasn’t the means of acquisition. Only one museum currently has the funding to contend for the best pieces–the J. Paul Getty Museum, founded by the billionaire oilman. The art may disappear into private hands, but its transfer will disseminate once static fortunes into the hands of various investors, collectors, and occasionally the artist.

Q. The passage supplies information for answering which of the following questions?

Detailed Solution for GMAT Focus Edition Mock test - 1 - Question 28

This is a DETAIL question. To answer detail questions, use the passage map to find the appropriate paragraph to find the relevant details, then go back and research each answer choice to avoid distortions and other common wrong answer traps.

To answer this open-ended detail question, examine each choice. However, your passage map tells you that you can limit your research of all five options to Paragraph 3.

Choice (A): do you know who owned van Gogh’s Portrait of Dr. Gachet prior to its purchase by Saito? No, you are only told that it was on loan to a museum. You are told nothing about Saito’s exhibiting the portrait; in fact, it is implied that he did not exhibit it at all. Therefore, choice (B) is wrong.

Choice (C) is incorrect because there is no mention of anyone proposing to purchase the portrait from Saito.

Choice (D) is the correct answer.

You are told that the representative of the Van Gogh Museum admitted that “he had no legal redress”; this means that no legal action could be threatened. The passage offers us no information that would answer the question posed in choice (E): the author reports both the threat to destroy the portrait, and Saito’s dismissal of that threat as a “joke,” but the author does not tell you what to believe about this point.

GMAT Focus Edition Mock test - 1 - Question 29

A prominent investor who holds a large stake in the Burton Tool Company has recently claimed that the company is mismanaged, citing as evidence the company's failure to slow down production in response to a recent rise in its inventory of finished products. It is doubtful whether an investor's sniping at management can ever be anything other than counterproductive, but in this case, it is clearly not justified. It is true that an increased inventory of finished products often indicates that production is outstripping demand, but in Burton's case it indicates no such thing. Rather, the increase in inventory is entirely attributable to products that have already been assigned to orders received from customers.

Q. In the argument given, the two boldfaced portions play which of the following roles? (underline)

Detailed Solution for GMAT Focus Edition Mock test - 1 - Question 29

Premise 1: An investor has claimed that Burton tool company is mismanaged. The evidence presented by him is failure to slow down production & increasing inventory of finished goods.

Premise 2: The author of the argument says in most cases the investors snipping has a positive effect on the strategy of a company, however in this case the investors claims are not justified. The author as a whole opposes the investors claim & provides a common view applicable for the situation and how it is not applicable in burton tool company's case

Conclusion: The argument as a whole thinks that the Burton tool company has valid reason for the inventory.

Analysis: The company strategy for holding inventory is justified with two supporting reasons by the author. The argument as a whole opposes the investors claims & justifies the case for burton tool company with supporting evidences.

(A) The first states a generalization that underlies the position that the argument as a whole opposes; the second provides evidence to show that the generalization does not apply in the case at issue. - Correct. The first provides a common view in favor of the opposing argument & the second proves that the first is not applicable in this case.

(B) The first states a generalization that underlies the position that the argument as a whole opposes; the second clarifies the meaning of a specific phrase as it is used in that generalization. - The second does not clarify meaning of first phrase. Incorrect

(C) The first provides evidence to support the conclusion of the argument as a whole; the second is evidence that has been used to support the position that the argument as a whole opposes. Incorrect. The first does not provide support for the conclusion

(D) The first provides evidence to support the conclusion of the argument as a whole; the second states that conclusion. - The second is not the conclusion. Incorrect.

(E) The first and the second each provide evidence against the position that the argument as a whole opposes. - Evidence is not against the position that is opposed. Incorrect.

GMAT Focus Edition Mock test - 1 - Question 30

A child learning to play the piano will not succeed unless the child has an instrument at home on which to practice. However, good-quality pianos, whether new or secondhand, are costly. Buying one is justified only if the child has the necessary talent and perseverance, which is precisely what one cannot know in advance. Consequently, parents should buy an inexpensive secondhand instrument at first and upgrade if and when the child's ability and inclination are proven.

Q. Which of the following, if true, casts the most serious doubt on the course of action recommended for parents?

Detailed Solution for GMAT Focus Edition Mock test - 1 - Question 30

The argument talks about 'child' (say 3 - 12 yrs). Option (D) talks about 'very young' which implies a little child (say 3 - 5 yrs). Since very young will be a subset of child, most of my interest in the option is already lost. It is like saying "some children will..." and that is usually irrelevant.
Secondly it uses "beginners" which means the stage at which they have the second hand cheap instrument. They may show promise but will stop suddenly. Well, all the more reason to wait and watch with a temporary instrument. If you begin with an expensive one, it will go waste.
If we know that kids show progress initially and then stand still, we would buy a cheaper instrument and see whether they are showing continuous progress to buy an expensive one later.

View more questions
Information about GMAT Focus Edition Mock test - 1 Page
In this test you can find the Exam questions for GMAT Focus Edition Mock test - 1 solved & explained in the simplest way possible. Besides giving Questions and answers for GMAT Focus Edition Mock test - 1, EduRev gives you an ample number of Online tests for practice

Top Courses for GMAT

Download as PDF

Top Courses for GMAT